Difference between revisions of "2023 AMC 10B Problems/Problem 18"

(Solution)
(redirect)
(Tag: New redirect)
 
(7 intermediate revisions by 5 users not shown)
Line 1: Line 1:
== Solution (Guess and check + Contrapositive)==
+
#redirect[[2023 AMC 12B Problems/Problem 15]]
<math>I.</math>  Try <math>a=3,b=5 => c = 17\cdot15</math> which makes <math>\textbf{I}</math> false.
 
At this point, we can rule out answer A,B,C.
 
 
 
<math>II.</math> A => B or C. equiv. ~B AND ~C => ~A.
 
Let a = 14, b=15 (statisfying ~B and ~C). => C = 2*210. which is ~A.
 
 
 
<math>II</math> is true.
 
 
 
So the answer is E.
 
<math>\boxed{\textbf{(E) } II \text{ and } III \text{only}.}</math>
 
~Technodoggo
 

Latest revision as of 20:45, 15 November 2023